2020-2021/TST/DM/2102_DM2/TST3/tpl_2102_DM2.tex

227 lines
8.5 KiB
TeX

\documentclass[a5paper,10pt]{article}
\usepackage{myXsim}
\usepackage{tasks}
% Title Page
\title{DM2 \hfill \Var{Nom}}
\tribe{TST}
\date{\hfillÀ render pour le Mercredi 24 février}
\xsimsetup{
solution/print = false
}
\begin{document}
\maketitle
\begin{exercise}[subtitle={Loi binomiale}]
%- set p = round(random(), 2)
Trois personnes s'apprêtent à passer le portique de sécurité. On suppose que pour chaque personne la probabilité que le portique sonne est égale à $\Var{p}$.
Soit $X$ la variable aléatoire donnant le nombre de personnes faisant sonner le portique, parmi les 3 personnes de ce groupe.
\begin{enumerate}
\item Tracer l'arbre représentant le situation.
\item Justifier que $X$ suit une loi binomiale dont on précisera les paramètres.
\item Quelle est la probabilité qu'une seule personne fasse sonner le portique?
\item Calculer puis interpréter les probabilités suivantes
\[
P(X = 0) \qquad \qquad P(X \geq 2)
\]
\item Calculer l'espérance de $X$ et interpréter le résultat.
\end{enumerate}
\end{exercise}
\begin{solution}
%- set q = round(1-p, 2)
\begin{enumerate}
\item
\begin{tikzpicture}[sloped]
\node {.}
child {node {$0$}
child {node {$0$}
child {node {$0$}
edge from parent
node[above] {\Var{q}}
}
child {node {$1$}
edge from parent
node[above] {\Var{p}}
}
edge from parent
node[above] {\Var{q}}
}
child[missing] {}
child {node {$1$}
child {node {$0$}
edge from parent
node[above] {\Var{q}}
}
child {node {$1$}
edge from parent
node[above] {\Var{p}}
}
edge from parent
node[above] {\Var{q}}
}
edge from parent
node[above] {\Var{q}}
}
child[missing] {}
child[missing] {}
child[missing] {}
child { node {$1$}
child {node {$0$}
child {node {$0$}
edge from parent
node[above] {\Var{q}}
}
child {node {$1$}
edge from parent
node[above] {\Var{p}}
}
edge from parent
node[above] {\Var{q}}
}
child[missing] {}
child {node {$1$}
child {node {$0$}
edge from parent
node[above] {\Var{q}}
}
child {node {$1$}
edge from parent
node[above] {\Var{p}}
}
edge from parent
node[above] {\Var{q}}
}
edge from parent
node[above] {\Var{p}}
} ;
\end{tikzpicture}
\item Chaque personne a 2 possibilités (1: fait sonner ou 2: ne fait pas sonner) et l'on fait passer 3 personnes ce qui correspond à une répétition identique et aléatoire. On peut donc modéliser la situation par une loi binomiale.
\[
X \sim \mathcal{B}(3; \Var{0.76})
\]
%- set p0 = round(q**3, 3)
%- set p1 = round(3*p*q**2, 3)
%- set p2 = round(3*p**2*q, 3)
%- set p3 = round(p**3, 3)
\item Probabilité qu'une seule personne fasse sonner le portique. On voit qu'il y a 3 branches qui correspondent à cette situation dont
\[
P(X = 1) = 3 \times \Var{p}^1 \times \Var{q}^2 \approx \Var{p1}
\]
\item
\[
P(X = 0) = \Var{q}^3 \approx \Var{p0}
\]
\[
P(X \geq 2) = P(X = 2) + P(X = 3) = 3 \times \Var{p}^2 \times \Var{q}^1 + \Var{p}^3 \approx \Var{round(p2 + p3, 3)}
\]
\item Il faut d'abord tracer le tableau résumant la loi de probabilité:
\begin{center}
\begin{tabular}{|c|*{4}{c|}}
\hline
Valeur & 0 & 1 & 2 & 3 \\
\hline
Probabilité & $\Var{p0}$ & $\Var{p1}$ & $\Var{p2}$ &$\Var{p3}$ \\
\hline
\end{tabular}
\end{center}
On peut alors calculer l'espérance
%- set E = round(3*p, 3)
\[
E[X] = 0 \times \Var{p0} + 1 \times \Var{p1} + 2 \times \Var{p2} + 3 \times \Var{p3} = \Var{E}
\]
On peut donc estimer qu'il y aura en moyenne $\Var{E}$ personnes qui feront sonner le portique sur les 3 personnes.
\end{enumerate}
\end{solution}
\begin{exercise}[subtitle={Équation puissance}]
Résoudre les équations et inéquations suivantes
\begin{multicols}{2}
\begin{enumerate}
%- set a1 = randint(1, 50)
\item $10^x = \Var{a1}$
%- set a2 = randint(1, 50)
%- set b2 = randint(2, 20)
\item $\Var{b2}^x = \Var{a2}$
%- set a3 = randint(2, 50)
%- set b3 = round(random(), 2)
\item $\Var{b3}^x \leq \Var{a3}$
%- set a4 = randint(2, 50)
%- set b4 = round(random(), 2)
%- set c4 = randint(2, 10)
\item $\Var{c4} \times \Var{b4}^x = \Var{a4}$
\end{enumerate}
\end{multicols}
\end{exercise}
\begin{solution}
Les solutions ci-dessous ne sont pas justifiée car l'ordinateur ne sait pas faire. Par contre, vous vous devez savoir justifier vos réponses!
\begin{enumerate}
\item $x = \log(\Var{a1})$
\item $x = \frac{\log(\Var{a2})}{\log(\Var{b2})}$
\item Il faut faire attention quand on divise par un log car ce dernier peut être négatif ce qui est le cas ici. Il faut donc pense à changer le sens de l'inégalité.
$x \geq \frac{\log(\Var{a3})}{\log(\Var{b3})}$
\item Il faut penser à faire la division à par $\Var{c4}$ avant d'utiliser le log car sinon, on ne peut pas utiliser la formule $\log(a^n) = n\times \log(a)$.
$x = \frac{\log(\Var{round(a4/c4, 2)})}{\log(\Var{b4})}$
\end{enumerate}
\end{solution}
\begin{exercise}[subtitle={Étude de fonctions}]
%- set a_ = randint(-10, 10)
%- set a = a_*3
%- set x1 = randint(0, 50)
%- set x2 = randint(-20, 20)
%- set b = randint(-50, 50)
%- set f = Polynom.from_coefficients([b, a*x1*x2, -a*(x1+x2)/2, a_])
Soit $f(x) = \Var{f}$ une fonction définie sur $\R$.
\begin{enumerate}
\item Calculer $f'(x)$ la dérivée de $f(x)$.
\item Calculer $f'(\Var{x1})$ et $f'(\Var{x2})$.
\item En déduire une forme factorisée de $f'(x)$.
\item Étudier le signe de $f'(x)$ et en déduire les variations de $f(x)$.
\item Est-ce que la fonction $f(x)$ admet un maximum ou un minimum? Si oui, calculer sa valeur.
\end{enumerate}
\end{exercise}
\begin{solution}
\begin{enumerate}
%- set fp = f.differentiate()
\item Dérivée de $f(x)$: $f'(x) = \Var{fp}$
\item
\begin{align*}
f'(\Var{x1}) &= \Var{fp(x1).explain() |join('\\\\&= ')}
\end{align*}
\begin{align*}
f'(\Var{x2}) &= \Var{fp(x2).explain() |join('\\\\&= ')}
\end{align*}
Donc $x = \Var{x1}$ et $x=\Var{x2}$ sont des racines de $f'(x) = \Var{fp}$.
\item On en déduit la forme factorisée suivante
\[
f'(x) = \Var{a} (x - \Var{x1})(x-\Var{x2})
\]
\item Pas de correction disponible
\item À causes des branches extérieurs, la fonction $f(x)$ n'a pas de maximum ou de minimum.
\end{enumerate}
\end{solution}
%\printsolutionstype{exercise}
\end{document}
%%% Local Variables:
%%% mode: latex
%%% TeX-master: "master"
%%% End: